Difference between revisions of "1996 AHSME Problems/Problem 5"

(Created page with "==See also== {{AHSME box|year=1996|num-b=4|num-a=6}}")
 
 
(One intermediate revision by one other user not shown)
Line 1: Line 1:
 +
==Problem==
 +
 +
Given that <math> 0 < a < b < c < d </math>, which of the following is the largest?
 +
 +
<math> \text{(A)}\  \frac{a+b}{c+d} \qquad\text{(B)}\ \frac{a+d}{b+c} \qquad\text{(C)}\  \frac{b+c}{a+d} \qquad\text{(D)}\  \frac{b+d}{a+c} \qquad\text{(E)}\ \frac{c+d}{a+b} </math>
 +
 +
==Solution 1==
 +
 +
Assuming that one of the above fractions is indeed always the largest, try plugging in <math>a=1, b=2, c=3, d=4</math>, since those are valid values for the variables given the constraints of the problem.  The options become:
 +
 +
<math> \text{(A)}\  \frac{1+2}{3+4} \qquad\text{(B)}\ \frac{1+4}{2+3} \qquad\text{(C)}\  \frac{2+3}{1+4} \qquad\text{(D)}\  \frac{2+4}{1+3} \qquad\text{(E)}\ \frac{3+4}{1+2} </math>
 +
 +
Simplified, the options are <math>\frac{3}{7}, 1, 1, \frac{3}{2}, \frac{7}{3}</math>, respectively.  Since <math>\frac{7}{3}</math> is the only option that is greater than <math>2</math>, the answer is <math>\boxed{E}</math>.
 +
 +
==Solution 2==
 +
 +
To make a fraction large, you want the largest possible numerator, and the smallest possible denominator.  Since <math>c</math> and <math>d</math> are the two largest numbers, they should go in the numerator as a sum.  Since <math>a</math> and <math>b</math> are the smallest numbers, they should go in the denominator as a sum.  Thus, the answer is <math>\boxed{E}</math>.
 +
 +
You can compare option <math>E</math> with every other fraction:  all numerators are smaller than <math>E</math>'s numerator, and all denominators are larger than <math>E</math>'s denominator.
 +
 
==See also==
 
==See also==
 
{{AHSME box|year=1996|num-b=4|num-a=6}}
 
{{AHSME box|year=1996|num-b=4|num-a=6}}
 +
{{MAA Notice}}

Latest revision as of 14:07, 5 July 2013

Problem

Given that $0 < a < b < c < d$, which of the following is the largest?

$\text{(A)}\  \frac{a+b}{c+d} \qquad\text{(B)}\ \frac{a+d}{b+c} \qquad\text{(C)}\  \frac{b+c}{a+d} \qquad\text{(D)}\  \frac{b+d}{a+c} \qquad\text{(E)}\ \frac{c+d}{a+b}$

Solution 1

Assuming that one of the above fractions is indeed always the largest, try plugging in $a=1, b=2, c=3, d=4$, since those are valid values for the variables given the constraints of the problem. The options become:

$\text{(A)}\  \frac{1+2}{3+4} \qquad\text{(B)}\ \frac{1+4}{2+3} \qquad\text{(C)}\  \frac{2+3}{1+4} \qquad\text{(D)}\  \frac{2+4}{1+3} \qquad\text{(E)}\ \frac{3+4}{1+2}$

Simplified, the options are $\frac{3}{7}, 1, 1, \frac{3}{2}, \frac{7}{3}$, respectively. Since $\frac{7}{3}$ is the only option that is greater than $2$, the answer is $\boxed{E}$.

Solution 2

To make a fraction large, you want the largest possible numerator, and the smallest possible denominator. Since $c$ and $d$ are the two largest numbers, they should go in the numerator as a sum. Since $a$ and $b$ are the smallest numbers, they should go in the denominator as a sum. Thus, the answer is $\boxed{E}$.

You can compare option $E$ with every other fraction: all numerators are smaller than $E$'s numerator, and all denominators are larger than $E$'s denominator.

See also

1996 AHSME (ProblemsAnswer KeyResources)
Preceded by
Problem 4
Followed by
Problem 6
1 2 3 4 5 6 7 8 9 10 11 12 13 14 15 16 17 18 19 20 21 22 23 24 25 26 27 28 29 30
All AHSME Problems and Solutions

The problems on this page are copyrighted by the Mathematical Association of America's American Mathematics Competitions. AMC logo.png